2^n<n! beweisen

Neue Frage »

Sia Auf diesen Beitrag antworten »
2^n<n! beweisen
Meine Frage:
2^n<n! Beweisen


Kann mir irgendjemand dabei helfen?!?! Wär echt nett!

lg, Sia=)

Meine Ideen:
mit Induktion halt.... Der anfang ist klar, aber dann der Induktionsschritt also von n-->n+1
das ei setzten und wie muss ich dann umfprmen, damit ich nachher auf mein gewünschtes ergebnis komm?
Iorek Auf diesen Beitrag antworten »

Dabei helfen kann dir bestimmt einer, es wird dir aber keiner die Lösung vorkauen. Gib genau an wo du hängst, wie weit kommst du im Induktionsschritt, bei welcher Umformung hängst du?
Sia Auf diesen Beitrag antworten »

ich bin beim anfang des Induktionsschritts:

2^(n+1) = 2^n x2 < (n+1)!

Ich hab keine ahnung was ich mit der Fakultät machen soll bzw. wie ich mit der umgehen soll...?!?! Deswegen komm ich nicht weiter...
Iorek Auf diesen Beitrag antworten »

Erstmal sollten wir mit dem Induktionsanfang was machen, für welche n gilt denn die Behauptung überhaupt?
klarsoweit Auf diesen Beitrag antworten »

Zitat:
Original von Sia
Ich hab keine ahnung was ich mit der Fakultät machen soll

Weißt du wenigstens was n! bedeutet? Dann sollte auch klar sein, wie man (n+1)! anders darstellen kann.
Schnurz Auf diesen Beitrag antworten »

2^1 = 2 > 1! = 1

2^2 = 4 > 2! = 2

2^3 = 8 > 3! = 6

Damit wäre bewiesen, dass 2^n < n! falsch ist.
 
 
Mazze Auf diesen Beitrag antworten »

Zitat:
2^1 = 2 > 1! = 1 2^2 = 4 > 2! = 2 2^3 = 8 > 3! = 6 Damit wäre bewiesen, dass 2^n < n! falsch ist.


Die Ungleichung



gilt erst ab n = 4.
Kühlkiste Auf diesen Beitrag antworten »

Zitat:
Original von Schnurz
2^1 = 2 > 1! = 1

2^2 = 4 > 2! = 2

2^3 = 8 > 3! = 6

Damit wäre bewiesen, dass 2^n < n! falsch ist.


Genauer: Damit wäre bewiesen, dass 2^n < n! für n < 4 falsch ist.

Das dumme ist nur, dass die Aussage für n > 3 richtig ist und mit einer nahezu tirivialen Induktion bewiesen werden kann, was Du nun einfach mal tun solltest.
klarsoweit Auf diesen Beitrag antworten »

Zitat:
Original von Kühlkiste
was Du nun einfach mal tun solltest.

Aber eigentlich nicht Schnurz, sondern Sia. Augenzwinkern
Kühlkiste Auf diesen Beitrag antworten »

Zitat:
Original von klarsoweit
Zitat:
Original von Kühlkiste
was Du nun einfach mal tun solltest.

Aber eigentlich nicht Schnurz, sondern Sia. Augenzwinkern


Ach so, ja natürlich.

Allerdings würde es Schnurz wohl auch nicht schaden ...
Mystic Auf diesen Beitrag antworten »

Zitat:
Original von klarsoweit
[Aber eigentlich nicht Schnurz, sondern Sia. Augenzwinkern


Ja, dem Schnurz ist die Aufgabe "schnurzegal", es sei denn, Schnurz ist in Wahrheit Sia... Big Laugh

@topic

Es wundert mich, dass sich hier alle in die vollständige Induktion "verbeißen", obwohl die dem Threadersteller ganz offensichtlich ein Buch mit sieben Siegeln ist... Man könnte ja z.B. auch die äquivalente Behauptung



zeigen, wozu man im wesentlichen nur die Beziehung



und die Monotonie von ln x benötigt...
AD Auf diesen Beitrag antworten »

Letztlich ist das aber auch eine verkappte Induktion, wobei man sich das mit Logarithmus sparen kann. Augenzwinkern

für alle
Kühlkiste Auf diesen Beitrag antworten »

Zitat:
Original von Mystic
Es wundert mich, dass sich hier alle in die vollständige Induktion "verbeißen", obwohl die dem Threadersteller ganz offensichtlich ein Buch mit sieben Siegeln ist... Man könnte ja z.B. auch...


einfach die Produktdarstellung von bemühen, an der sich die Aussage direkt ablesen lässt.
Mystic Auf diesen Beitrag antworten »

Zitat:
Original von Kühlkiste
einfach die Produktdarstellung von bemühen, an der sich die Aussage direkt ablesen lässt.

Sehe ich jetzt nicht unmittelbar auf eine andere Weise, als dies schon Arthur oder auch ich (mit Hilfe von , allerdings mit dem tasächlich unnötigen Übergang zu Logarithmen) bereits dargelegt haben...
Kühlkiste Auf diesen Beitrag antworten »

Zitat:
Original von Mystic
Zitat:
Original von Kühlkiste
einfach die Produktdarstellung von bemühen, an der sich die Aussage direkt ablesen lässt.

Sehe ich jetzt nicht unmittelbar auf eine andere Weise, als dies schon Arthur oder auch ich (mit Hilfe von , allerdings mit dem tasächlich unnötigen Übergang zu Logarithmen) bereits dargelegt haben...


Gemeint hatte ich folgendes:



und bezogen hat sich das ausschließlich auf Deinen Post von 10:29h.
Mystic Auf diesen Beitrag antworten »

Ist wahrscheinlich jetzt ein Streit um des "Kaisers Bart", aber ich denke, es ist im Prinzip die gleiche Idee wie in meinem Beweis, was man natürlich jetzt noch viel besser sehen würde, wenn man den - wie schon Arthur bemerkt hat - unnötigen Übergang zu Logarithmen wegläßt und stattdessen gleich direkt über das Produkt argumentiert...
gonnabphd Auf diesen Beitrag antworten »

Spam

Die Helfer streiten sich um den einfachsten Beweis und Sia hat sich schon abgesetzt... Big Laugh
Mystic Auf diesen Beitrag antworten »

Zitat:
Original von gonnabphd
Spam

Die Helfer streiten sich um den einfachsten Beweis und Sia hat sich schon abgesetzt... Big Laugh


Ja, der Preis dafür ist in meinen Augen bereits längst an Arthur ergangen, da sein Beweis unter rein formalen Gesichtspunkten doch noch eine Spur schöner ist als der auch sehr kurze und ansonsten ziemlich ähnliche von Kühlkiste...

Ich möchte aber vielleicht doch noch kurz erwähnen, warum ich überhaupt auf die Idee gekommen bin, zu logarithmieren, indem ich nämlich ursprünglich die Abschätzung



im Auge hatte, die ja für n >2e, also dann einen direkten Beweis liefert, wobei man die Fälle n=4 und n=5 noch manuell überprüfen müßte... Ich habe aber dann diese Idee als zu kompliziert verworfen, bin aber bei den Logarithmen irgendwie "hängengeblieben"... unglücklich

Ja, und dann hatte ich auch noch die Idee



zu zeigen, was ja, wie man nach Multiplikation mit n! sofort sieht, eine äquivalente Behauptung ist.. Auch das sieht man mit diversen Abschätzungen der Summanden der linken Summe sehr schnell, ist aber dann auch nicht wirklich einfacher...

Immerhin sind das substanziell andere Beweisvarianten und für mich, obwohl zugegebenermaßen weit komplizierter, tausendmal attraktiver als die üblichen stupiden Beweise mit vollständiger Induktion... Just my two cents... Augenzwinkern
Neue Frage »
Antworten »



Verwandte Themen

Die Beliebtesten »
Die Größten »
Die Neuesten »